3
$\begingroup$

I am currently going through Shimura's paper on half-integer weight modular forms. I would like to understand given a 𝑞-expansion of half-integral weight modular forms of arbitrary level and character, how to compute the effect of the Hecke operator and the Atkin-Lehner operator/Fricke involution in SAGE/Magma. I am a beginner in SAGE/Magma and
The only documentation I found is related to computing basis of weight k/2 and character chi.

If anyone has any sources on or explanations as to how to compute how q-expansions are transformed under the action of Hecke operators and Atkin-Lehner/Fricke involution, it would be greatly appreciated!

$\endgroup$
1
  • $\begingroup$ I'd be very surprised if this were implemented, it's already pretty difficult to compute the action of Atkin--Lehner operators in integer weight (there are no simple formulas for the action in terms of q-expansions). $\endgroup$ Commented Jul 5, 2023 at 12:52

1 Answer 1

7
$\begingroup$

This can be done using PARI/GP, which can deal with spaces of modular forms of half-integral weight. Given a modular form $f$ of weight $k$ (possibly half-integral), the command mfslashexpansion can compute the $q$-expansion of $f |_k g$ for any $g \in \mathrm{GL}_2^+(\mathbf{Q})$. This relies on a floating-point method but works well in practice. One of the ideas is to multiply your half-integral weight modular form by the weight $1/2$ modular form $\theta$, whose behaviour under the Atkin-Lehner involution is known. This reduces to the case of integral weight, which is still difficult, but one other idea is to write the form as a linear combination of pairwise products of Eisenstein series. The behaviour of Eisenstein series under the Atkin-Lehner involution is known, which gives the result.

You can look at PARI/GP's users manual (see the section Modular forms). If you want to know the mathematics behind the algorithms, you can read this article by Belabas and Cohen.

$\endgroup$
2
  • $\begingroup$ Thank you very much! I will look into it. I had some idea about Magma/Sage that is why I was trying to compute it by using either of these. I do not have any idea about PARI/GP. I will try to use the manual of it to use the suggestions you provided. Thank you again. $\endgroup$ Commented Jul 17, 2023 at 1:31
  • 1
    $\begingroup$ Maybe the tutorial on modular forms can be helpful: pari.math.u-bordeaux.fr/pub/pari/manuals/2.15.1/tutorial-mf.pdf $\endgroup$ Commented Jul 18, 2023 at 6:33

You must log in to answer this question.

Start asking to get answers

Find the answer to your question by asking.

Ask question

Explore related questions

See similar questions with these tags.